18.1.1 Несобственные интегралы I рода (интегралы по неограниченным промежуткам)

Пусть функция $f$ задана на промежутке $[a, +\infty)$, где $a \in R$, и интегрируема по Риману на каждом отрезке $[a, \xi)$, где $a \lt \xi \lt +\infty$. Выражение $\int_a^{+\infty} f(x) dx$ называют несобственным интегралом I рода. Если существует $\lim\limits_{\xi\to +\infty}\int_a^\xi f(x) dx$ то этот несобственный интеграл называют сходящимся, а его значение полагают равным:
$$\int_{a}^{\infty}f(x)dx=\lim_{\xi \to +\infty}\int_{a}^{\xi}f(x)dx.$$
Если же не существует конечного предела, то несобственный интеграл называют расходящимся.

Аналогично определяется несобственный интеграл:
$$\int_{-\infty}^{a}f(x)dx = \lim_{\eta\to -\infty}\int_{\eta}^{a}f(x)dx.$$

Пусть теперь функция $f$ задана на всей действительной прямой и интегрируема по Риману на любом отрезке $\left[\eta, \xi\right]$, где $ -\infty \lt \eta \lt \xi \lt +\infty.$
Если существует конечный двойной предел $\lim\limits_{\substack{\xi\to +\infty \\ \eta\to-\infty}}\int_\eta^\xi f(x) dx$,то несобственный интеграл $\int_{-\infty}^{+\infty} f(x) dx$ называется сходящимся, а его значение полагают равным $$\int_{-\infty}^{+\infty}f(x)dx = \lim_{\substack{\xi\to +\infty\\ \eta\to-\infty}}\int_{\eta}^{\xi}f(x)dx.$$

Утверждение. Сходимость интеграла $\int_{-\infty}^{+\infty}f(x)dx$ равносильна тому, что сходятся оба интеграла $\int_{a}^{+\infty}f(x)dx$ и $\int_{-\infty}^{a}f(x)dx$, причем имеет место равенство $$\int_{-\infty}^{+\infty}f(x)dx = \int_{-\infty}^{a}f(x)dx + \int_{a}^{+\infty}f(x)dx$$
где a – произвольное действительное число.

Пусть при некотором $a \in R$ интегралы $\int_{a}^{+\infty}f(x)dx$ и $\int_{-\infty}^{a}f(x)dx$ сходятся. Тогда для $-\infty \lt \eta \lt \xi \lt +\infty$ будем иметь
$$\int_{\eta}^{\xi}f(x)dx = \int_{\eta}^{a}f(x)dx + \int_{a}^{\xi}f(x)dx$$
Отсюда, переходя к пределам при $\xi → +\infty$ и $\eta → -\infty$, получаем
$$\lim_{\substack{\xi\to +\infty \\ \eta\to-\infty}}\int_{\eta}^{\xi}f(x)dx = \lim_{\substack{\xi\to +\infty \\ \eta\to-\infty}}\int_{\eta}^{a}f(x)dx + \lim_{\substack{\xi\to +\infty \\ \eta\to-\infty}}\int_{a}^{\xi}f(x)dx=\\
= \int_{-\infty}^{a}f(x)dx + \int_{a}^{+\infty}f(x)dx$$
т. е. интеграл $\int_{-\infty}^{+\infty}f(x)dx$ сходится и для него справедливо равенство $\int_{-\infty}^{+\infty}f(x)dx = \int_{-\infty}^{a}f(x)dx + \int_{a}^{+\infty}f(x)dx$.

Для доказательства обратного утверждения зафиксируем произвольное $a \in R$ и предположим, что существует
$$\int_{-\infty}^{+\infty}f(x)dx=\lim_{\substack{\xi\to +\infty \\ \eta\to-\infty}}\int_{\eta}^{\xi}f(x)dx.$$
Тогда, в силу критерия Коши существования двойного предела, отсюда
следует, что для любого $ \varepsilon\gt 0$ найдется такое $A$, что для любых $\xi^{\prime}, \xi^{\prime\prime} \gt A$ и для любых $\eta^{\prime},\eta^{\prime\prime}\lt −A$ справедливо неравенство
$$\left|\displaystyle\int_{\eta^{\prime}}^{\xi^{\prime}}f(x)dx — \int_{\eta^{\prime\prime}}^{\xi^{\prime\prime}}f(x)dx\right|\lt \varepsilon $$
Зафиксируем $\varepsilon \gt 0$ и найдем такое $A$. Можем считать, что $A\gt|a|$. Выберем $\eta=\eta^{\prime}=\eta^{\prime\prime}\lt −A$ и $\xi^{\prime}, \xi^{\prime\prime}\gt A$. Тогда получим
$$\left|\displaystyle\int_{\xi^{\prime}}^{\xi^{\prime\prime}}f(x)dx\right| = \left|\displaystyle\int_{\eta}^{\xi^{\prime}}f(x)dx — \int_{\eta}^{\xi^{\prime\prime}}f(x)dx\right|\lt \varepsilon, $$
т. е. выполнено условие критерия Коши существования предела
$$\lim_{\xi\to +\infty}\int_{a}^{\xi}f(x)dx.$$
Отсюда следует, что интеграл $\int_{a}^{+\infty}f(x)dx$ сходится. Аналогично получаем, что и интеграл $\int_{-\infty}^{a}f(x)dx$ также сходится. Имеем
$$\int_{-\infty}^{a}f(x)dx + \int_{a}^{+\infty}f(x)dx = \lim_{\eta\to -\infty}\int_{\eta}^{a}f(x)dx + \lim_{\xi\to +\infty}\int_{a}^{\xi}f(x)dx =\\
= \lim_{\substack{\xi\to +\infty \\ \eta\to-\infty}}\left(\displaystyle\int_{\eta}^{a}f(x)dx + \int_{a}^{\xi}f(x)dx\right) = \lim_{\substack{\xi\to +\infty \\ \eta\to-\infty}}\int_{\eta}^{\xi}f(x)dx = \int_{-\infty}^{+\infty}f(x)dx$$ Последний предел существует в силу условия, а выражение справа не
зависит от $a$. Тем самым доказано $\int_{-\infty}^{+\infty}f(x)dx = \int_{-\infty}^{a}f(x)dx + \int_{a}^{+\infty}f(x)dx$ для любого $a \in R$.

Пример 1. Вычислим $$\int_0^{+\infty}\frac{dx}{1+x^2} = \lim_{\xi\to +\infty}\int_{0}^{\xi}\frac{dx}{1+x^2}=\lim_{\xi\to +\infty} {\mathrm {arctg}}\,x\bigg|_0^{\xi} = \lim_{\xi\to +\infty}{\mathrm {arctg}}\,x=\frac{\pi}{2}.$$

Пример 2. Несобственный интеграл $\int_0^{+\infty}\sin x dx.$ расходится. В самом деле, $$\int_0^{\xi}\sin x dx =-\cos x \bigg|_0^{\xi}= 1-cos {\xi} $$ не имеет предела.

Примеры решения задач

Пример 1

Вычислить $\int_0^{+\infty}e^{-px}dx.$

\underline {Решение:}

$$\int_0^{+\infty}e^{-px}dx= -\frac{1}{p}e^{-px}\bigg|_0^{+\infty}=-\frac{1}{p}\lim_{x\to +\infty}(e^{-px}-1)= \begin{cases}
\frac{1}{p}, \text{если $p \gt 0$;} \\
+\infty, \text{если $p\lt 0$.}
\end{cases}$$ При $p \gt 0 \lim\limits_{x\to +\infty}e^{-px}= \lim\limits_{x\to +\infty}\frac{1}{e^{px}}=0$, так как $e^{px}\to+\infty$ при $x\to+\infty.$ При $p\lt 0 \lim\limits_{x\to +\infty}e^{-px} = +\infty.$

Таким образом, интеграл $\int_0^{+\infty}e^{-px}dx$ сходится при $p \gt 0$ и расходится при $p\lt 0.$

[свернуть]

Пример 2

При каких значениях показателя $\lambda \gt 0$ существует несобственный интеграл $\int_a^{+\infty}\frac{dx}{x^\lambda}, (a\gt 0).$

\underline {Решение:}

Пусть $\lambda\neq1$, тогда $$\int_a^{\xi}\frac{dx}{x^\lambda}=\frac{1}{1-\lambda}x^{1-\lambda}\bigg|_a^\xi=\frac{1}{1-\lambda} (\xi^{1-\lambda} — a^{1-\lambda}).$$
Это выражение при $\xi\to+\infty$ имеет предел $\infty$ ( $\lambda \lt 1$) или конечное число $\frac{1}{1-\lambda} a^{1-\lambda}$ ($\lambda \gt 1$).

Если $\lambda=1$, имеем $$\int_a^{\xi}\frac{dx}{x}=\ln(x)\bigg|_a^\xi=\ln(\xi)-\ln(a)$$ и при $\xi\to+\infty$ в пределе получается $+\infty$. Таким образом, интеграл $\int_a^{+\infty}\frac{dx}{x^\lambda}$ при $\lambda\gt 1$ сходится (и равен $\frac{1}{1-\lambda} a^{1-\lambda}$), а при $\lambda\leq 1$ расходится.

[свернуть]

Пример 3

Вычислить $\int_{-\infty}^{+\infty}\frac{dx}{1+x^2}.$

\underline {Решение:}

$$\int_{-\infty}^{+\infty}\frac{dx}{1+x^2}=\lim\limits_{x\to{+\infty}} {\mathrm {arctg}}\,x -\lim\limits_{x\to{-\infty}} {\mathrm {arctg}}\,x = \frac{\pi}{2} -(-\frac{\pi}{2})=\pi.$$

Интеграл $\int_{-\infty}^{+\infty}\frac{dx}{1+x^2}$ сходится и равен $\pi$.

[свернуть]

Несобственные интегралы по неограниченным промежуткам

Для закрепления пройденного материала предлагается пройти тест.

Литература

  1. Коляда В.И.,Кореновский А.А. Курс лекций по математическому анализу / В.И.Коляда.-Одесса: Изд-во «Астропринт», 2010. т.2. -С.102-105.
  2. Каплан И.А. Практические занятия по высшей математике / И.А.Каплан. -Харьков: Изд-во Харьковского университета, 1967. ч.3. -С.760-761.
  3. Фихтенгольц Г.М. Курс дифференциального и интегрального исчисления/ Г.М.Фихтенгольц -Москва: Изд-во «Наука», 1969. т.2. -С.553.
  4. Лысенко З.М. Конспект лекций по математическому анализу.